A split highway has a number of lanes for traffic. For traffic going in one direction, the radius for the inside of the curve is half the radius for the outside. One car, car A, travels on the inside while another car, car B, travels at equal speed on the outside of the curve. Which statement about resultant accelerations of the cars is correct

Answers

Answer 1

Answer:

The correct one is that the force on B is half of the force on A

Explanation:

Because radius for the inside of the curve is half the radius for the outside and Car A travels on the inside while car B, travels at equal speed on the outside of the curve. Thus force on B will be half on A


Related Questions

Alice and Bob are bicycle-racing, on a two-mile track. Bob rides the first mile at 50 mph,
and the second mile at 71 mph. Calculate the time in minutes) that he travels the two
miles.

Answers

Answer:

2.04 minutes

Explanation:

He travels the first mile at 50 mph so:

[tex]60/50=1.2[/tex]

so it takes 1.2 minutes to travel each mile when going 50 miles per hour

and for the second mile he travels at 71 mph so:

[tex]60/71=0.84[/tex] (this answer is rounded)

so it takes him 0.84 minutes to travels each mile while going 71 mph.

If you ad them together his total times while biking two miles was 2.04 minutes.

What is the toy's total energy at any point of its motion? Express your answer with the appropriate units.]

Answers

The complete question is as follows:

A 0.150-kg toy is undergoing SHM on the end of a horizontal spring with force constant k = 300 N/m. When the toy is 0.0120 m from its equilibrium position, it is observed to have a speed of 0.400 m/s.

Answer:

The correct answer is 0.034 J.

Explanation:

Given :

mass of the toy is m = 0.15 kg.  

The force constant of restoring force k = 300 Nm⁻¹

When the position of the toy from the equilibrium is x = 0.012m, then the  

speed of the toy vx = 0.4m s

The total mechanical energy in SHM is given by  

E=  1/2 (mv²+ kx²) = 1/2 kA²

(here, m = mass of the object, vx = velocity, k = force constant  

of restoring force, and A = amplitude of SHM.)

Hence by substituting the numerical values in equation 1, we get  

[tex]E= \frac{1}{2} (0.15* 0.4) + \frac{1}{2} (300* 0.012)[/tex]

= 0.034 J

Thus, the correct answer is 0.034 J.

X-rays with wavelength of 2.0 nm scatter from a NaCL crystal with plane spacing of 0.281 nm. Find the scattering angle for the second order maxima.

Answers

Explanation:

Wavelength of x-rays = 2 nm

Plane spacing, d = 0.281 nm

It is assumed to find the scattering angle for second order maxima.

For 2nd order, Bragg's law is given by :

[tex]2dsin\theta =n\lambda[/tex]

For second order, n = 2

[tex]sin\theta =\frac{n\lambda}{2d} \\Sin\theta =\frac{2\times2}{2\times0.281} \\\theta =\arcsin (7.14)[/tex]

Here, θ is not defined. Also, the wavelength of x-rays is more than the plane spacing. It means that it cannot produce any diffraction maximum.

You want to create an electric field vector E = < 0, 5 104, 0> N/C at location < 0, 0, 0>. Where would you place a proton to produce this field at the origin?

Answers

Answer:

1.696 × 10^(-7) m on the y axis.

Explanation:

We are given the electric field as;

E = < 0, 5 × 10⁴, 0> N/C

This is written in (x, y, z) co-ordinates. So it means that it lies on the y-axis.

So,

E = 5 × 10⁴ N/C in the y direction.

Formula for Electric field is;

E = kq/r²

where;

k is a constant with a value of 8.99 x 10^(9) N.m²/C²

q is charge on the proton = 1.6 × 10^(-19) C

r is the distance

Thus, making r the subject gives;

r = √(kq/E)

Plugging in the relevant values gives;

r = √(8.99 × 10^(9) × 1.6 × 10^(-19)/(5 × 10⁴))

r = 1.696 × 10^(-7) m on the y axis.

The proton to produce this field at the origin should be put on [tex]1.696 \times 10^{(-7)} [/tex]m on the y axis.

The calculation is as follows:

We are given the electric field as;

[tex]E = < 0, 5 \times 10^{4}, 0> N/C[/tex]

This is written in (x, y, z) co-ordinates. So it means that it lies on the y-axis.

So,

[tex]E = 5 \times 10^{4} N/C[/tex] in the y direction.

The Formula for Electric field is;

E = kq/r²

where;

k is a constant with a value of 8.99 x 10^(9) N.m²/C²

q is charge on the proton = 1.6 × 10^(-19) C

r is the distance

Thus, making r the subject gives;

r = √(kq/E)

Now Plugging in the relevant values gives;

r = √(8.99 × 10^(9) × 1.6 × 10^(-19)/(5 × 10⁴))

r = 1.696 × 10^(-7) m on the y axis.

learn more: https://brainly.com/question/4626564?referrer=searchResults

Find the tension between the 4 kg and the 6 kg blocks once the system is released from rest and the 9 kg block accelerates downward.

Answers

Answer:

Tension on the string is 48 N

Explanation:

Mass of A = 6 kg

mass of B = 4 kg

If the 9 kg accelerates downwards then the equation for the 6 kg mass side of the pulley will be

6g - T = 6a    ....1

where

g is acceleration due to gravity = 10 m/s^2

a is the acceleration of the body

T is the tension in the string.

For the 4 kg mass side of the string, we have

T - 4g = 4a    ....2

transposing, we have

T = 4a + 4g    ....3

substitute equation 3 in equation 1, we have

6g - (4a + 4g) = 6a

6g - 4a - 4g = 6a

6g - 4g = 6a + 4a

2g = 10a

but g = 10 m/s^

2 x 10 = 10a

20 = 10a

acceleration a = 20/10 = 2 m/s^2

substitute value of a in equation 1

6g - T = 6a

6(10) - T = 6(2)

60 -T = 12

60 - 12 = T

T = 48 N

If the steam in the engine is able to raise a 30 kg piston to a height of 0.68 m, how much work has been done by the steam?

Answers

Answer:

199.92joules

Explanation:

workdone = Mgh

mass x height x acceleration due to gravity

The x-component of vector R is Rx = −28.2 units and its y-component is Ry = 19.6 units. What are its magnitude and direction? Give the direction as an angle measured counterclockwise from the +x-direction.

Answers

Answer:

Explanation:

Rx = -28.2 units

Ry = 19.6 units

magnitude of R = √  [( - 28.2 )² + ( 19.6 ) ]

= √ ( 795.24 + 384.16 )

= 34.34 units

If θ  be the angle measured counterclockwise from the +x-direction

Tanθ = 19.6 / - 28.2 = -0.695

θ = 180 - 34.8

= 145.2° .

When a circuit is arranged in parallel
A. There are multiple paths that electrons can take through the circuit, and it is possible for the electron to pass through one circuit component but not another.
B. There is only one path the electrons can take through the circuit, and they must pass through all circuit components

Answers

Answer:

A. There are multiple paths that electrons can take through the circuit, and it is possible for the electron to pass through one circuit component but not another.

Explanation:

Parallel arrangement of components in an electric circuit puts different parts of the circuit on different branches. In a parallel connection, there are multiple paths for the electrons to take, and it is possible for electrons to pass through on circuit component without going through another. This is the reason why If there is a break in one branch of the circuit, electrons can still flow in other branches, and the same reason why one bulb going off in your home does prevent the other components in your home from coming on (your home is wired in a parallel electric circuit).

the effect on any force depends on what two things

Answers

Answer: mass and distance

Explanation: Two major factors, mass and distance, affect the strength of gravitational force on an object. You witness the first factor in everyday life - more massive objects are heavier. The second factor, distance, is less familiar, as it takes a distance of many thousands of kilometers to significantly weaken the pull of Earth's gravity.

Answer:

The factor of a force depends on two things:

1. the amount of a force applied. The more the force applied the greater the effect.

2.the area on which the force is applied. The smaller the area the force is applied on, the greater the effect.

plz mark brainliest

Acceleration of 1.5 ms expressed in km /hr2? ​

Answers

you’re answer is 5 because !! :)

What operation do you apply to the position function of a particle to compute the particles velocity

Answers

Answer:

the derivative with respect to time

Explanation:

This is an exercise in kinematics, where the velocity is defined as a function of the position of a body of the form

            v = dx/dt

where v is the velocity of the body, x is the position that we assume is a continuous and differentiable function.

The function written in the equation is the derivative with respect to time

what magnetic field strength will allow the electron to pass between the plates without being deflected

Answers

Complete Question

An electron travels with speed 2.0 X 10^7 m/s between two parallel charged plates. The plates are separated by 2.0 cm and are charged by a 400V battery. What magnetic field strength and direction will allow the electron to pass between the plates without being deflected.?

Answer:

The value  [tex]B = 0.001 \ T[/tex]

Explanation:

From the question we are told that

   The  speed of the electron is  [tex]v = 2.0 *10^{7} \ m/s[/tex]

    The voltage  is  [tex]V = 400 \ V[/tex]

     The distance of separation is  [tex]d= 2.0 \ cm = 0.02 \ m[/tex]

Generally the electric field is mathematically represented as

       [tex]E = \frac{V}{d }[/tex]

=>    [tex]E = \frac{400}{0.02 }[/tex]

=>     [tex]E = 20000 V/m[/tex]

Generally for the electron to pass without deflection then

     [tex]F_B = F_E[/tex]

Where [tex]F_B[/tex] is the maximum magnetic force acting on the electron which is mathematically represented as

       [tex]F_B = evBsin (\theta )[/tex]

Since we are considering maximum magnetic field then [tex]\theta = 90^o[/tex]

=>   [tex]F_B = evB[/tex]

And  

    [tex]F_E[/tex] is the electric force on the electron which is mathematically represented as

       [tex]F_E = e * E[/tex]

Here e is the charge on the electron with the value [tex]e = 1.60 *10^{-19} \ C[/tex]

=>   [tex]F_E = 1.60*10^{-19} * 20000[/tex]

=>      [tex]F_E = 3.2*10^{-15} \ N[/tex]

So

    [tex]3.2*10^{-15} = q* v B[/tex]

=>    [tex]B = \frac{3.2*10^{-15}}{ 2.0*10^7 * 1.6*10^{-19}}[/tex]

=>     [tex]B = 0.001 \ T[/tex]

It takes Serina 4.44 hours to drive to school. Her route is 15 km long. What is Serina’s average speed on her drive to school?

Answers

Answer: 3.38 km per hour

Explanation:

A toy police car moves across the room in 10 seconds. The distance the car traveled is
450 centimeters. At what speed did the car travel?

Answers

Answer:

45cm/s

Explanation:

If the distance traveled is 450 centimeters we can assume the speed is 45cm/s by taking the distance and dividing by the time, in this case that is 450 and 10

so our answer would be 45cm/s

when would a swimmer have the most potential energy due to her position

Answers

Answer: A

Explanation:

A swimmer have the most potential energy due to her position.When he is sitting on the highest diving board at the pool.

What is potential energy?

The potential energy is the due to the virtue of the position and the height. The unit for the potential energy is joule.

The potential energy is found as;

[tex]\rm PE = mgh[/tex]

The potential energy is mainly depending upon the height of the object.

When the swimmer is at the highest position, the height is maximum. Therefore, the potential energy is also maximum.

Hence, a swimmer have the most potential energy due to her position.When he is sitting on the highest diving board at the pool.

To learn more about the potential energy, refer to the link;

https://brainly.com/question/24284560

#SPJ2

Robert pushes the box to the left at constant speed. In doing so, Robert does ______ work on the box. A. positive B. negative C. zero

Answers

Answer:

He does postive work

Explanation:

Because W= F xS sinစ

So since work is done such that Force and distance have the Sam direction

Thus စ=0

Then work is positive in value

How much power is used by a hair dryer if it does 40,000 J of work in 40 seconds?

Answers

Answer:

40000÷40=1000 joules is required to work in 40 seconds

define polarization in reference in the simple cell​

Answers

Answer:polarization pertains to the act or process of producing a positive electrical charge and a negative electrical charge such that between a nerve cell internal electrical charge..................

Explanation:

A-0.06 C charge that moves downward is in a uniform electric field with a strength of 200 N/C. What is the
magnitude and direction of the force on the charge?
O 12 Nup
O 12 N down
O 3333 Nup
O 3333 N down

Answers

Answer:

its B

Explanation:

i just took the test

The correct option is B. The magnitude of the force on the charge is 12N and the direction is downward.

The electric field is a region of space where electric force can be felt. The formula for calculating the magnitude of a force in an electric field is expressed as:

[tex]\overline F=qE[/tex] where:

q is the charge measured in CoulombsE is the electric field intensity measured in N/C

Given the following parameters:

q = -0.06C

E = 200N/C

Substitute the given parameters into the formula as shown:

[tex]\overline F = -0.06 * 200\\\overline F = -12N\\[/tex]

Since the modulus of the force is in the negative sense, hence the direction will be downward.

Learn more here: https://brainly.com/question/22735564

Effect of thunder storm ​

Answers

Answer:

Some of the most severe weather occurs when a single thunderstorm affects one location for an extended time. Thunderstorms can bring heavy rains (which can cause flash flooding), strong winds, hail, lightning, and tornadoes. Severe thunderstorms can cause extensive damage to homes and property.

Explanation:

Which of the following does NOT cause acceleration?

A change in the magnitude of the velocity
A change in the direction of the velocity
Constant speed
Increasing speed
Decreasing spped

Answers

Answer:

Constant speed

Explanation:

Acceleration is a change of magnitude and (or) direction of velocity, it will cause increase or decrease, or changing of the direction of the velocity.

Find the value of x in the triangle shown below.
9
C
3

Answers

add the image so we can answer it for you, 621

A particle moves along a line so that its velocity at time t is ( | |. Find the displacement of the particle during the time period .

Answers

Complete question is;

A particle moves along a line so that its velocity at time t is

v(t) = t² − t − 6

(measured in meters per second).

(a) Find the displacement of the particle during the time period 1 ⩽ t ⩽ 4.

(b) Find the distance traveled during this time period.

Answer:

A) -4.5 m

B) 10.17 m

Explanation:

We are given;

v(t) = t² − t − 6

We know that v = ds/dt

Thus,

S = v dt

s(4) - s(1) = (4,1)∫t² − t − 6 dt

= (4,1)[(t³/3) - (t²/2) - 6t]

= ((4³/3) - (4²/2) - 6(4)) - ((1³/3) - (1²/2) - 6(1))

= 64/3 - 8 - 24 - 1/3 + 1/2 + 6

= -4.5 m

B) Since v(t) = t² − t − 6, then factorizing we can write it as;

v(t) = (t + 2)(t - 3)

Thus, v(t) ⩽ 0 at the interval (1, 3) and v(t) ≥ 0 at the interval (3, 4)

Thus;

-x1 = (1, 3)∫t² − t − 6 dt

-x1 = (1,3)(t³/3 - t²/2 - 6t)

-x1 = (3³/3 - 3²/2 - 6(3)) - (1³/3 - 1²/2 - 6(1))

-x1 = 9 - 9/2 - 18 - 1/3 + 1/2 + 6

-x1 = -3 - 4 - 1/3

-x1 = -22/3 m

x1 = 22/3 m

x2 = (3, 4)∫t² − t − 6 dt

x2 = (3, 4)(t³/3 - t²/2 - 6t)

x2 = ((4³/3) - (4²/2) - 6(4)) - (3³/3 - 3²/2 - 6(3))

x2 = 64/3 - 8 - 24 - 9 + 9/2 + 18

x2 = -23 + 64/3 + 9/2

x2 = (-138 + 128 + 27)/6

x2 = 17/6 m

Thus, total distance = x1 + x2 = 22/3 + 17/6 = 10.17m

A kitten runs 24 x 10⁻² m away from its master in a straight line in 12.6 s, and then run halfway back in one-third the time. Calculate its average velocity. ​

Answers

Answer:

1.43 × 10^-2 m/s

Explanation:

Given the following :

Distance covered away from it's master = D1 = 24×10^-2 m

Time taken to cover the distance = t1 = 12.6s

Then runs halfway back D2 =

D1/ 2 = (24× 10^-2m) / 2 = 12 ×10^-2m

Time taken =T2 = T1/3 = 12.6/3 = 4.2s

Since the kitten ran in a straight line, the it is linear :

Average speed

Linear Average Velocity:

Change in distance / change in time

(D1 - D2) / (T2 - T1)

(24 - 12)×10^-2 / (12.6 - 4.2)

(12 × 10^-2)m / 8.4s

1.4285 × 10^-2

= 1.43 × 10^-2m/s

An electron moving parallel to the x axis has an initial speed of 4.10 106 m/s at the origin. Its speed is reduced to 1.76 105 m/s at the point x = 2.00 cm.

Required:
a. Calculate the electric potential difference between the origin and that point.
b. Which point is at the higher potential?

Answers

Answer:

a

  [tex]V = -47.65 N/C[/tex]

b

At the origin

Explanation:

From the question we are told that

   The  initial speed is  [tex]v_1 = 4.10 *10^{6} \ m/s[/tex]

    The  speed at (x = 2.00 cm) is  [tex]v_f = 1.76 *10^{5} \ m/s[/tex]

   

Generally the electric potential difference is mathematically represented as

    [tex]V = \frac{W}{q}[/tex]

Here W is the  work-done which is mathematically represented as

    [tex]W = K_f - K_i[/tex]

Here  [tex]K_f[/tex] is the kinetic energy at  x =  2.00 cm mathematically expressed as

        [tex]K_f = \frac{1}{2} * m* v^2_f[/tex]

and  

       [tex]K_i[/tex] is the kinetic energy at origin  mathematically expressed as

        [tex]K_f = \frac{1}{2} * m* v^2_i[/tex]

So

    [tex]V = \frac{1}{q} [ \frac{1}{2} mv_f^2 - \frac{1}{2} mv_i^2][/tex]

    [tex]V = \frac{m}{2q} [v_f^2 - v_i^2][/tex]

Here m is the mass of electron with value [tex]m = 9.1*10^{-31} \ kg[/tex]

         q  is the charge on the electron with value  [tex]q = 1.60*10^{-19} \ C[/tex]

So

     [tex]V = \frac{(9.1 *10^{-31})}{2(1.60*10^{-19})} [(1.76*10^{5})^2 - (4.10*10^{6})^2][/tex]

     [tex]V = -47.65 N/C[/tex]

So given that the difference is negative then potential is higher at the origin

   

A cannonball falls at a rate of about 10 m/s2 due to gravity. It has a mass of 200 kg. What is the total force of the
cannonball?

Answers

[tex]\star[/tex][tex]\blue\star[/tex]

[tex]\blue\star[/tex]Givenmass = 200kgacceleration = 10m\s^2

Star coding

[tex]\blue\star[/tex]As we know that force = mass × acceleration

F = m×aF = 200 ×10F = 2000N

[tex]\blue\star[/tex]so mate here is ur ans force is equal to 2000N.

Hope it helps

If a baseball travels a distance of 4 meters in 5 seconds, what is its average speed

Answers

Answer:

speed=d/t.

s=4/5=0.8m/s.

A 7600 kg satellite is in a circular orbit around Earth at a height of 2300 km above Earth's surface. What is this satellite's speed

Answers

Answer:

6779.7m/s

Explanation:

Using

GMm/(Re +h)² = mv²/ (Re+h)

So making v subject we have

V= √GM/Re+h

So

V = √ 6.67*10^-11 x 5.97*10^24/(6371+2300)*10^3

V= 6779.7m/s

Note h = height of satellite

Re= radius of the earth

M = mass of the earth

As the frequency of the ac voltage across an inductor approaches zero, the inductive reactance of that coil Group of answer choices

Answers

Answer:

The inductive reactance also approaches zero

Explanation:

This is because from the relation

XL = 2πfL

Meaning frequency f and inductive reactance XL are directly proportional so as one approaches zero so does the other

A variable is defined as anything that

Answers

Answer : Quantity or quality that varies like manipulating a variable to see what will happen to another variable
Other Questions
Point C is on line segment \overline{BD} BD . Given CD=x,CD=x, BC=5x-5,BC=5x5, and BD=2x+7,BD=2x+7, determine the numerical length of \overline{CD}. CD . The question is in the image below. Amelia bought 20 chicken wings for $50.00. How much would it cost for 14 wings? why do we accelerate forward when we run? Need HELP ASAP drivers education write a paragraph discussing what things you and your passenger can do to safely exit your vehicle The area of the face of a cube is 9 square centimeters. Use 9 of these cubes to make a rectangular prism. What is the volume of the rectangular prism William read in his textbook that he should avoid bouncing checks like the plague. He learned the hard way just how true this is. When he wrote a $7.50 check to Subway he only had $6.00 in his checking account and his check bounced. He was astounded at the cost of this first bounced check. If this cost was the same as the national average, what did William pay for bouncing his first check Maana ella ________ si aprob el examen de ingreso. what language does Doktor Pannwitz speak to Levi? Alex spends 2/3 of his paycheck p on food and $3 lose change on coffee how much did he spend Point A is the terminal point on the unit circle associated with real number pi/5. What are the coordinates of this point? Please solve no.7 for me with full steps wrong answer will get reported A right triangle has a hypotenuse of 37 and a leg of 10. What is the second leg of the triangle? On September 1, 2021, Blue Co., issued $1,600,000 of its 10% bonds at 98 plus accrued interest. The bonds are dated June 1, 2021 and have an effective interest rate of 11%. Interest is payable semiannually on June 1 and December 1. At the time of issuance, Blue would receive cash of: The value of x when h=4 is The cross-price elasticity of demand for coffee and coffee-cream is likely to be Group of answer choices What was the "penny-farthing"?O A. A steam powered railroadO B. An electric streetcarO C. A bicycle with a huge wheel in frontO D. A bicycle with two wheels the same size Observe o as pirmides etrias analisando as transformaes ocorridas na demografia desse pas da Amrica Central. Em seguida realiza um pequeno texto descrevendo as transformaes ocorridas na estrutura etria, urbanizao e distribuio da populao no PEA. Portland cement concrete (PCC) has a density of 150 lb/ft3. How many kilograms of PCC are required to cast a cylindrical column 1.15 m in diameter and 7.8 m in height? (1 kg = 2.205 lb, 1 inch = 2.54 cm) Which sentence from the paragraph best supports the idea that this paragraph belongs in the middle of Sora's essay? That day after practice, I was frustrated and stomped home o In fact, I'm embarrassed to say I sulked for the rest of the day Then my stubborn streak kicked in, and I decided that failure wasn't an option. I practiced regularly until the day came when I was able to consistently hit the ball